if a sample of 57 swimmers is taken from a population of 135 swimmers, the population mean is the mean of how many swimmers' times?

Answers

Answer 1

Answer:

135

Step-by-step explanation:

A P E X


Related Questions

A ball is thrown straight up from a cliff. The function f(x)= -4.9t^ + 17t +19 describes the height of the ball, in meters, as a function of time, t, in seconds. What is the maximum height of the ball? At what time is the height reached? Round your answer to one decimal place.

Answers

Answer:

  33.7 m at 1.7 seconds

Step-by-step explanation:

For quadratic ax^2+bx+c, the line of symmetry (x-coordinate of the vertex) is ...

  -b/(2a)

For your quadratic, the vertex (highest point) is reached at time ...

  t = -(17)/(2(-4.9)) = 17/9.8 ≈ 1.7 . . . . seconds

__

The height at that time is ...

  f(17/9.8) = (-4.9(17/9.8) +17)(17/9.8) +19 = 289/19.6 +19 ≈ 33.7 . . . meters

_____

Comment on the function evaluation

We have used the "Horner form" of the function to make evaluation easier.

  f(t) = (-4.9t +17)t +19

How many boys are there in an introductory geology course if 360 students are enrolled and there are five boys to every seven girls?

Answers

Answer:150 branliest

Step-by-step explanation:

Answer:

150

Step-by-step explanation:

360/(5+7)=30

30*5=150

If you roll a die three times, what is the probability of rolling three ONES?

(Give your answer as a decimal, rounded to the nearest thousandth. That is, rounded to three decimal places.)

Answers

Answer:

0.004

Step-by-step explanation:

The probability of rolling any number is 1/6 so that times 3 is 1/216 that as a decimal i 0.004

Please help ASAP! I will mark Brainliest! Please READ the question THEN answer CORRECTLY! No guessing!

Answers

Answer:

C.  [tex]\frac{\sqrt{5} }{8}[/tex]

Step-by-step explanation:

This expression can be rewritten as [tex]\frac{\sqrt{5} }{\sqrt{64} }[/tex].

Since the square root of 5 is prime and does not have any perfect square factors, it cannot be simplified. However, the square root of 64 is equal to 8, so our final simplified radical expression would be [tex]\frac{\sqrt{5} }{8}[/tex], which is option C.

HOPE THIS HELPED! :)

Answer:

C is the answer

Step-by-step explanation:

You have to rewrite as [tex]\sqrt{5}[/tex]/[tex]\sqrt{64}[/tex]. Then you have to simplify the denominator which is 8. That is how you get your answer.

Hope this helps.

Ms Davis is doing an activity with her statistic students where she gives them a 20 question multiple top choice test and they know none of the answers. Students need to guess on every question and each question has 5 possible choices, 1 of the which is correct.

What is the mean and standard deviation of the number of questions that each student gets correct?

Answers

Answer:

The mean and standard deviation of the number of questions that each student gets correct are 4 and 1.789 respectively.

Step-by-step explanation:

Let the random variable X be defined as the number of correct answers marked by a student.

It is provided that each question has 5 possible choices, 1 of the which is correct.

Then the probability of marking thee correct option is:

[tex]P(X)=\frac{1}{5}=0.20[/tex]

There are a total of n = 20 questions to be answered.

As the students does not the answer to any question, they would be guessing for each question. This implies that for a random question, all the five options has the equal probability of being correct and each of the five options can be correct independently from the other.

All these information above indicates that the random variable X follows a Binomial distribution with parameters n = 20 and p = 0.20.

The mean and standard deviation of a Binomial distribution are:

[tex]\mu=np\\\\\sigma=\sqrt{np(1-p)}[/tex]

Compute the mean and standard deviation of the random variable X as follows:

[tex]\mu=np=20\times 0.20=4\\\\\sigma=\sqrt{np(1-p)}=\sqrt{20\times 0.20\times(1-0.20)}=1.789[/tex]

Thus, the mean and standard deviation of the number of questions that each student gets correct are 4 and 1.789 respectively.

The mean will be "4" and the standard deviation will "1.789".

According to the question,

The probability of making 3 correct options will be:

→ [tex]P(X) = \frac{1}{5}[/tex]

            [tex]= 0.20[/tex]

Total number of questions,

n = 20

As we know,

The mean will be:

→ [tex]\mu = np[/tex]

By substituting the values, we get

     [tex]= 20\times 0.20[/tex]

     [tex]= 4[/tex]

and,

The standard deviation will be:

→ [tex]\sigma = \sqrt{np(1-p)}[/tex]

      [tex]= \sqrt{20\times 0.20\times (1-0.20)}[/tex]

      [tex]= 1.789[/tex]

Thus the responses above are correct.

Learn more about standard deviation here:

https://brainly.com/question/20896613

A sphere is inscribed in a cube. Explain the relationship between the surface areas of the two solid figures

Answers

Answer:

No one knows. Jk just go look at the other brainly links. the question has already been asked and answered so you can go see them there.

Step-by-step explanation:

:)

Find the greatest common factor of the
following monomials:
34c3 2c5

Answers

Answer:2c^2

Step-by-step explanation:

34c^3 and 2c^5

34c^3=2 x 16 x c x c^2

2c^5=2 x c^2 x c^3

Greatest common factor =2 x c^2

Greatest common factor =2c^2

A bag contains 10 Yellow, 4 Green and 7 Blue marbles. Find the following probabilies.
P(blue)

Answers

Answer:

1/3

Step-by-step explanation:

The total number of marbles

10+4+7 = 21

P(blue) = blue marbles / total marbles

            = 7 / 21

            = 1/3

c. two hundred thirty-one and five tenths - in decimal standard form​

Answers

Answer:

231.5

five tenths = 0.5

Standard form of a decimal 231.5

You need to be taller than 48" to ride the roller coaster. Which of the following inequalities shows the height Susan has to be to ride the roller coaster.

h < 48"

h > 48"

h ≤ 48"

h ≥ 48"

Answers

Answer:

> 48"

Step-by-step explanation:

> represents greater than 48"

Points X( -6, -4), Y( -2, -8), Z( 2,-4) are the vertices of a right triangle. What is the area of XYZ? Round to the nearest tenth.

Answers

Answer:

16

Step-by-step explanation:

distance between point x and z is 8 units

distance between point z and y is 4 units

8 * 4 = 32

formula for area of a triangle (l * w)/2

plugin: (8 * 4)/2 = 16

the midpoint of AB is point P at (-16,6) if point A is at (-10,8) what are the coordinates of point B?

Answers

Answer:

Denote B(x, y), we have:

-10 + x = 2 x (-16) => x = -22

  8 + y = 2 x 6 => y = 4

=> B(-22, 4)

Hope this helps!

:)

A grocery store has an average sales of $8000 per day. The store introduced several advertising campaigns in order to increase sales. To determine whether or not the advertising campaigns have been effective in increasing sales, a sample of 64 days of sales was selected. It was found that the average was $8300 per day. From past information, it is known that the standard deviation of the population is $1200. The p-value is

Answers

Answer:

The p-value of the test is 0.023.

Step-by-step explanation:

In this case we need to determine whether the addition of several advertising campaigns increased the sales or not.

The hypothesis can be defined as follows:

H₀: The stores average sales is $8000 per day, i.e. μ = 8000.

Hₐ: The stores average sales is more than $8000 per day, i.e. μ > 8000.

The information provided is:

 [tex]n=64\\\bar x=\$8300\\\sigma=\$1200[/tex]

As the population standard deviation is provided, we will use a z-test for single mean.

Compute the test statistic value as follows:

 [tex]z=\frac{\bar x-\mu}{\sigma/\sqrt{n}}=\frac{8300-8000}{1200/\sqrt{64}}=2[/tex]

The test statistic value is 2.

Decision rule:

If the p-value of the test is less than the significance level then the null hypothesis will be rejected.

Compute the p-value for the two-tailed test as follows:

 [tex]p-value=P(Z>2)\\=1-P(Z<2)\\=1-0.97725\\=0.02275\\\approx 0.023[/tex]

*Use a z-table for the probability.

The p-value of the test is 0.023.

The sum of the measures of the angles of a triangle is 180. The sum of the measures of the second and third angles is five times the measure of the first angle. The third angle is 14 more than the second. Let x, y, and z represent the measures of the first, second, and third angles, respectively. Find the measures of the three angles.

Answers

Answer:

[tex]x=30\\y=68\\z=82[/tex]

Step-by-step explanation:

x = measure of the first angle

y = measure of the second angle

z = measure of the third angle

The sum of the measures of the second and third (y+z) is five times the measure of the first angle (=5x)

[tex]y+z=5x[/tex]

The third angle is 14 more than the second

[tex]z=y+14[/tex]

And remember that the sum of these three angles must be equal to 180.

[tex]x+y+z=180[/tex]

Let's take these equations

[tex]y+z=5x\\z=y+14\\x+y+z=180[/tex]

If you take a look at the first equation, we have y+z = 5x and we have y+z in the third equation as well, we can replace that....

[tex]x+y+z=180\\x+(y+z)=180\\x+(5x)=180[/tex]

Distribute the + sign

[tex]x+5x=180[/tex]

Combine like terms;

[tex]6x=180[/tex]

Divide by 6.

[tex]x=\frac{180}{6}\\ x=30[/tex]

We have now defined that the measure of the first angle is 30º.

Let's take another equation... for example [tex]z=y+14[/tex]

I'm going to take this one because if I replace x and z in the third equation, all I'll have left will be y.

[tex]x+y+z=180\\30+y+(y+14)=180[/tex]

Distribute the + sign and Combine like terms;

[tex]30+y+y+14=180\\44+2y=180\\[/tex]

Subtract 44 to isolate 2y.

[tex]2y=180-44\\2y=136[/tex]

Now divide by 2.

[tex]y=\frac{136}{2}\\ y=68[/tex]

We already have the value of x and y. Once again, replacing this in the third equation will leave us with z to solve for.

[tex]x+y+z=180\\30+68+z=180\\98+z=180\\z=180-98\\z=82[/tex]

Really need help on this. I keep gettin it wrong please help!!!

Answers

Answer:

1436.76 m³

Step-by-step explanation:

Volume of sphere= 4/3 π r ³

V= 4/3(3.14)(7)³

V=4/3(3.14)(343)

V= 4310.26 / 3

V= 1436.76 m³

What is the solution to the system of equations?

3 x + 10 y = negative 47. 5 x minus 7 y = 40.
(1, –5)
(1, 5)
(–1, –5)
(–1, 5)

Answers

Answer:

  (1, –5)

Step-by-step explanation:

It is relatively easy to try the offered solutions to see what works.

(1, -5)

  3(1) +10(-5) = -47 . . . true

  5(1) -7(-5) = 40 . . . true

(1, -5) is the solution

_____

As a check, you can try some of the other choices:

(1, 5)

  3(1) +10(5) ≠ -47

(-1, -5)

  3(-1) +10(-5) ≠ -47

(-1, 5)

  3(-1) +10(5) ≠ -47

None of the other choices works in the first equation, so they're not the solution.

Answer:

(1,-5)

Step-by-step explanation:

1)get desmos

2)put in numbers

3)where they intersect is the answer.

easy as pie... oh wait pie ain't easy...

easy as ramen

2. Inflation is at a rate of 7% per year. Evan's favorite bread now costs $1.79. What did it cost 10 years ago? How long
before the cost of the bread doubles?​

Answers

Answer:

It cost $0.91 10 years ago.

It takes 10.24 years for the cost of bread to double.

Step-by-step explanation:

The equation for the price of bread after t years has the following format:

[tex]P(t) = P(0)(1+r)^{t}[/tex]

In which P(0) is the current price, and r is the inflation rate, as a decimal.

If we want to find the price for example, 10 years ago, we find P(-10).

Inflation is at a rate of 7% per year. Evan's favorite bread now costs $1.79.

This means that [tex]r = 0.07, P(0) = 1.79[/tex]. So

[tex]P(t) = P(0)(1+r)^{t}[/tex]

[tex]P(t) = 1.79(1+0.07)^{t}[/tex]

[tex]P(t) = 1.79(1.07)^{t}[/tex]

What did it cost 10 years ago?

[tex]P(-10) = 1.79(1.07)^{-10} = 0.91[/tex]

It cost $0.91 10 years ago.

How long before the cost of the bread doubles?

This is t for which P(t) = 2P(0) = 2*1.79. So

[tex]P(t) = 1.79(1.07)^{t}[/tex]

[tex]2*1.79 = 1.79(1.07)^{t}[/tex]

[tex](1.07)^{t} = 2[/tex]

[tex]\log{(1.07)^{t}} = \log{2}[/tex]

[tex]t\log{1.07} = \log{2}[/tex]

[tex]t = \frac{\log{2}}{\log{1.07}}[/tex]

[tex]t = 10.24[/tex]

It takes 10.24 years for the cost of bread to double.

Rewrite the percentage in the sentence below as a decimal.
The 20 overseas investors own 7.2% of the business.

Answers

Answer:

0.072

Step-by-step explanation:

Answer:0.072

Step-by-step explanation:

7.2% = 7.2/100

7.2 ➗ 100=0.072

simplify the expression (3k / 8)^2​

Answers

Answer:

9k/64^2 (9k^2/64^2)

Step-by-step explanation:

Answer:

9k^2/64

Step-by-step explanation:

3*3=9

k*k=k^2

8*8=64

Subtracting by adding up 65-39

Answers

Answer:

26

Step-by-step explanation:

In order to subtract by adding up 65-39, we need to add -39 to the value of 65. This can be rewritten in this way;

65+(-39)

The equation above is similar to the one given because the product of a minus and a plus sign will still give us back a minus sign.

on solving;

65+(-39) = 26

Ginger still has 40 percent of her book to read. If she has read 180 pages, how many pages does she still have to read?

Answers

Answer:

Ginger still has 40 percent of her book to read. If she has read 180 pages, the number of pages she still have to read:

N = 180 x (100 - 40)/40 = 270 pages

Hope this helps!

:)

Please help ASAP! I will mark Brainliest! Please answer CORRECTLY! No guessing!

Answers

Answer:

Option D: 5591.93

Step-by-step explanation:

The best way to understand this question is to apply the formula in an indirect manner;

P - Principle number, Starting Value

T - Time

I - Interest

Let us convert the interest into decimal form, such that 3.8% is shifted two decimal points to the right ⇒ 0.038. Now add 1 to this value to get ⇒ 1.038. By PEDMAS, you would first raise this value to the span of 3 years as such:

(1.038)^3 = 1.118386872.......

The final step would by to multiply the starting value (investment $) $ 5,000 by this continuing value of 1.118386872:

5,000(1.118386872) = 5591.93436 ⇒ Rounded to (About) 5591.93

Answer:D

Step-by-step explanation:

principal=p=$5000

Rate=r=3.8%

Time=n=3 years

amount=a

a=p(1+r/100)^n

a=5000(1+3.8/100)^3

a=5000(1+0.038)^3

a=5000(1.038)^3

a=5000 x 1.038 x 1.038 x 1.038

a=5591.93

a=$5591.93

If f(x)=9x+2, What is the value of the function when x=4?

If f(x)=9x+2, for what value of x is the value of the function 29?

Answers

Answer:

1) 38

2) 3

Step-by-step explanation:

f(x)=36+2

f(x)=38

29=9x+2

27=9x

x=3

Question: What is the value of the function at x=−2?

Answer: y=2

Step-by-step explanation: I took the test.

*THIS IS THE CORRECT ANSWER. PLEASE DON'T ANSWER 3. IT IS WRONG!*

Solve each equation.
13 = -2w

Answers

Answer:

w = -6.5

Step-by-step explanation:

We want to get rid of the coefficient of w so that we can solve for w. To do that, let's divide both sides of the equation by -2. When we do so, we get -13/2 = w. Hope this helps!

The management of Discount Furniture, a chain of discount furniture stores in the Northeast, designed an incentive plan for salespeople. To evaluate this innovative plan, 12 salespeople were selected at random, and their weekly incomes before and after the plan were recorded. Was there a significant increase in the typical salesperson’s weekly income due to the innovative incentive plan? Use the .05 significance level. Estimate the p-value, and interpret it

Answers

Answer:

Step-by-step explanation:

The question is incomplete. The complete question is

The management of Discount Furniture, a chain of discount furniture stores in the Northeast, designed an incentive plan for salespeople. To evaluate this innovative plan, 12 salespeople were selected at random, and their weekly incomes before and after the plan were recorded.

Salesperson Before After

Sid Mahone $320 $340

Carol Quick 290 285

Tom Jackson 421 475

Andy Jones 510 510

Jean Sloan 210 210

Jack Walker 402 500

Peg Mancuso 625 631

Anita Loma 560 560

John Cuso 360 365

Carl Utz 431 431

A. S. Kushner 506 525

Fern Lawton 505 619

Solution:

Corresponding income of salespersons before and after form matched pairs.

The data for the test are the differences between the income is salespersons.

μd = the​ income before minus their income after.

Bedore after diff

320 340 -20

290 285 5

421 475 - 54

510 510 0

210 210 0

402 500 - 98

625 631 -6

569 560 0

360 365 - 5

431 431 0

506 525 - 19

505 619 - 114

Sample mean, xd

= (- 20 + 5 - 54 + 0 + 0 - 98 - 6 + 0 - 5 + 0 + - 19 - 114)/12 = - 25.92

xd = - 25.92

Standard deviation = √(summation(x - mean)²/n

n = 12

Summation(x - mean)² = (- 20 + 25.92)^2 + (5 - 25.92)^2 + (- 54 + 25.92)^2+ (0 + 25.92)^2 + (0 + 25.92)^2 + ( - 98 + 25.92)^2 + ( - 6 + 25.92)^2 + (0 + 25.92)^2 + (- 5 + 25.92)^2 + (0 + 25.92)^2 + (- 19 + 25.92)^2 + (- 114 + 25.92)^2 = 17784.5168

Standard deviation = √(17784.5168/12

sd = 38.5

For the null hypothesis

H0: μd ≥ 0

For the alternative hypothesis

H1: μd < 0

1) The distribution is a students t. Therefore, degree of freedom, df = n - 1 = 12 - 1 = 11

2) The formula for determining the test statistic is

t = (xd - μd)/(sd/√n)

t = ( - 25.92- 0)/(38.5/√12)

t = - 2.33

3) We would determine the probability value by using the t test calculator.

p = 0.02

4) Assume alpha = 0.05

Since alpha, 0.05 > than the p value, 0.02, then we would reject the null hypothesis. We can conclude that at 5% significance level, there is a significant increase in the typical salesperson’s weekly income due to the innovative incentive plan

55 POINTS IF YOU GET RIGHT THEN GOATED AND BRAINIEST!!

Answers

Answer:

17 i would say it is D

18 i belive it is B

19 it is c

20 A

Carpetland carpet installers incur an average cost of $300 for each carpet installed. Joan Chin, the firm’s vice president, proposes a new procedure for installations, which she hopes will be more efficient. Joan plans to run a trial and hopes that the results of a trial period will enable her to conclude with a level of significance of 0.05 that the new procedure reduces the average cost to install a carpet. If Joan's hypothesis test results in a Type II error, what would this mean?

Answers

Answer:

A Type II error happens when the null hypothesis failed to be rejected, although it is false and the alternative hypothesis is true.

In this context, would be that the test does not give enough evidence to support the claim that the new procedure reduces the average cost, although it really does reduces it.

The new procedure is effective, but the sample does not give enough evidence.

Step-by-step explanation:

add 5 1/2 kg to 450g​

Answers

1kg = 1000g

51/2 kg= 51/2×1000=51×500=25500g

25500g+450g= 25950g

Answer:

[tex]5950g[/tex]

Step-by-step explanation:

[tex]5 \frac{1}{2} kg \\ = 5kg \: 500g \\ = 5500g[/tex]

Now let's add.

[tex] \: \: \: \: \: \: 5500g \\ + \: \: \: \: 450g \\ =5950 g[/tex]

So the correct answer is,

[tex] = 5950g = 5kg \: 950g[/tex]

hope this helps

brainliest appreciated

good luck! have a nice day!

Which fraction equals a repeating decimal?
30/50
13/25
5/30
13/10

Answers

Answer:

5/30

Step-by-step explanation:

5/30 = 1/6 =0.166666666...

The fraction equals a repeating decimal is 5/30.

What are decimals?

A decimal numeral system is the standard system for denoting integer and non-integer numbers. The way of denoting numbers in the decimal system is often referred to as decimal notation.

Now the given fractions are,

30/50

13/25

5/30

13/10

Converting them into decimals we get,

30/50 = 0.6

13/25 = 0.52

5/30 = 0.1666..

13/10 = 1.3

Thus the repeating decimal is 0.1666..

So, the fraction with repeating decimal is 5/30 = 0.1666..

Thus, the fraction equals a repeating decimal is 5/30.

To learn more about fraction :

brainly.com/question/1301963

#SPJ2

Find the surface area of the pyramid to the nearest whole number.

Answers

Answer:

i think its 248m^2

Step-by-step explanation:

Other Questions
find the area of the figure please slove the x in the digram below explain What is a intersection How does the current values in and out of the battery compare to the sum of currents going through the light bulbs?Note: This is for Parallel circuits. Please help ASAP for 10pts Please help with this An oak tree can be a home for animals like squirrels, birds, etc. as well as provide oxygen for the environment. This is an example of a _____________: the role an organism plays in the environment. Find the area. Will mark brainliest. select two values of x that are roots of this equation (picture included)x^2+2x-5=0 Anwar selects a playing card at random from the following 666 cards:\{{left brace 999 of hearts, 555 of spades, 666 of hearts, 222 of spades, 444 of hearts, 777 of hearts\}}right braceLet AAA be the event that Anwar selects an even-numbered card and BBB be the event that she chooses a heart.Which of the following statements are true? how are magnetic field lines like electric field lines? A. they always point away from the north poleB. they never connectC. they always point toward a negative poleD. they never cross Which of the following best describes how the narrator describes nature, especially the cold?AThe narrator describes the cold as a pervasive, almost personified force.BThe narrator describes nature constantly changing and unpredictable.CThe narrator describes nature, even the dog, as indifferent to the struggles of the man.DThe narrator describes the cold as merely an element that can be easily conquered by men and fire. Suppose a bicycle was coasting on a level surface, and there was no friction. What would happen to the bicycle?A.It would slow down until it finally came to a stop.OB. It would slow down but would never stop,c. It would speed up since there is no friction,OD. It would keep going at constant speed.Hurry please Is there a high or low standard of living? (Hint, look at how many babies are being born) High Low What does Keaton spend the most money on?Keaton's Monthly SpendingWhat does Keaton spend the least money on?FoodKeaton spends about as much money on rent as hedoes onRentClothesHygiene ProductsEntertainment A roof on a castle tower is shaped like a cone with a diameter of 12 feet and has a slant height of 13 feet one bundle of Shingles covers 32 ft. how many bundles should you buy to cover the roof? What is the surface area of the roof ? If the base-ten blocks shown are to be divided into 5 equal groups, what should be done first? help asap will mark brainliest Complete the statement with the choice that bestdescribes the relationship between the polynomialand its rational zeroes.A polynomial, P, has a leading coefficient of 1 anda constant term. The rational roots of Pare all What is the function of the esophagus? To chew food To churn food To move food to the mouth To move food to the stomach